Difference between revisions of "1994 AHSME Problems/Problem 24"

(Created page with "==Problem== A sample consisting of five observations has an arithmetic mean of <math>10</math> and a median of <math>12</math>. The smallest value that the range (largest observa...")
 
(Solution)
Line 4: Line 4:
 
<math> \textbf{(A)}\ 2 \qquad\textbf{(B)}\ 3 \qquad\textbf{(C)}\ 5 \qquad\textbf{(D)}\ 7 \qquad\textbf{(E)}\ 10 </math>
 
<math> \textbf{(A)}\ 2 \qquad\textbf{(B)}\ 3 \qquad\textbf{(C)}\ 5 \qquad\textbf{(D)}\ 7 \qquad\textbf{(E)}\ 10 </math>
 
==Solution==
 
==Solution==
 +
The minimum range occurs in the set <math>\{7,7,12,12,12\}</math>, so the answer is <math>\boxed{\textbf{(C)}\ 5}</math>

Revision as of 13:54, 15 February 2016

Problem

A sample consisting of five observations has an arithmetic mean of $10$ and a median of $12$. The smallest value that the range (largest observation minus smallest) can assume for such a sample is

$\textbf{(A)}\ 2 \qquad\textbf{(B)}\ 3 \qquad\textbf{(C)}\ 5 \qquad\textbf{(D)}\ 7 \qquad\textbf{(E)}\ 10$

Solution

The minimum range occurs in the set $\{7,7,12,12,12\}$, so the answer is $\boxed{\textbf{(C)}\ 5}$